Đến nội dung

Hình ảnh

số nghiệm nguyên dương của phương trình $a+b+c=n$

* * * * * 1 Bình chọn

  • Please log in to reply
Chủ đề này có 22 trả lời

#1
Nobodyv3

Nobodyv3

    Generating Functions Faithful

  • Thành viên
  • 937 Bài viết
1/ Tính số nghiệm nguyên dương của phương trình $a+b+c=n$ sao cho $a\geq b\geq c$ và $a<b+c.$
===========
Thà rót cho ta..... trăm nghìn chung... rượu độc ...miễn sao đừng bắt em làm toán!..hu hu...

#2
chanhquocnghiem

chanhquocnghiem

    Thiếu tá

  • Thành viên
  • 2494 Bài viết

1/ Tính số nghiệm nguyên dương của phương trình $a+b+c=n$ sao cho $a\geq b\geq c$ và $a<b+c.$

Đặt $n=12k+p$ ($0\leqslant p\leqslant 11$)

Nhận xét rằng :

+ $a$ chỉ có thể nhận giá trị từ $\left \lfloor \frac{n+2}{3} \right \rfloor$ đến $\left \lfloor \frac{n-1}{2} \right \rfloor$, từ đó suy ra $a$ chỉ có thể nhận $2k$ giá trị nếu $p\in \left \{ 0,1,2,4 \right \}$, nhận 2k+1 giá trị nếu $p\in\left \{ 3,5,6,7,8,10 \right \}$ và nhận 2k+2 giá trị nếu $p\in\left \{ 9,11 \right \}$

+ Nếu gọi số nghiệm của pt $b+c=n-a$ là $m$ thì khi $a$ tăng thêm $2$ đơn vị thì $m$ tăng thêm $3$.

+ Khi $p\in\left \{ 0,3,6,9 \right \}$ và $a$ lần lượt nhận giá trị $\left \lfloor \frac{n+2}{3} \right \rfloor$ và $\left \lfloor \frac{n+5}{3} \right \rfloor$ thì $m$ nhận giá trị $1$ và $2$

+ Khi $p\in\left \{ 1,4,7,10 \right \}$ và $a$ lần lượt nhận giá trị $\left \lfloor \frac{n+2}{3} \right \rfloor$ và $\left \lfloor \frac{n+5}{3} \right \rfloor$ thì $m$ nhận giá trị $2$ và $3$

+ Khi $p\in\left \{ 2,5,8,11 \right \}$ và $a$ lần lượt nhận giá trị $\left \lfloor \frac{n+2}{3} \right \rfloor$ và $\left \lfloor \frac{n+5}{3} \right \rfloor$ thì $m$ nhận giá trị $1$ và $3$

Từ các nhận xét trên, dễ dàng tính được tổng số nghiệm nguyên dương $s$ thỏa mãn các điều kiện đề bài :

$s=\left\{\begin{matrix}3k^2\ neu\ p=0\\3k^2+2k\ neu\ p\in\left \{ 1,4 \right \}\\3k^2+k\ neu\ p=2\\3k^2+3k+1\ neu\ p\in\left \{ 3,6 \right \}\\3k^2+4k+1\ neu\ p\in\left \{ 5,8 \right \}\\3k^2+5k+2\ neu\ p\in\left \{ 7,10 \right \}\\3k^2+6k+3\ neu\ p=9\\3k^2+7k+4\ neu\ p=11 \end{matrix}\right.$


Bài viết đã được chỉnh sửa nội dung bởi chanhquocnghiem: 03-01-2023 - 19:37

...

Ðêm nay tiễn đưa

Giây phút cuối vẫn còn tay ấm tay
Mai sẽ thấm cơn lạnh khi gió lay
Và những lúc mưa gọi thương nhớ đầy ...

 

http://www.wolframal...-15)(x^2-8x+12)


#3
Nobodyv3

Nobodyv3

    Generating Functions Faithful

  • Thành viên
  • 937 Bài viết

Đặt $n=12k+p$ ($0\leqslant p\leqslant 11$)
Nhận xét rằng :
+ $a$ chỉ có thể nhận giá trị từ $\left \lfloor \frac{n+2}{3} \right \rfloor$ đến $\left \lfloor \frac{n-1}{2} \right \rfloor$, từ đó suy ra $a$ chỉ có thể nhận $2k$ giá trị nếu $p\in \left \{ 0,1,2,4 \right \}$, nhận 2k+1 giá trị nếu $p\in\left \{ 3,5,6,7,8,10 \right \}$ và nhận 2k+2 giá trị nếu $p\in\left \{ 9,11 \right \}$
+ Nếu gọi số nghiệm của pt $b+c=n-a$ là $m$ thì khi $a$ tăng thêm $2$ đơn vị thì $m$ tăng thêm $3$.
+ Khi $p\in\left \{ 0,3,6,9 \right \}$ và $a$ lần lượt nhận giá trị $\left \lfloor \frac{n+2}{3} \right \rfloor$ và $\left \lfloor \frac{n+5}{3} \right \rfloor$ thì $m$ nhận giá trị $1$ và $2$
+ Khi $p\in\left \{ 1,4,7,10 \right \}$ và $a$ lần lượt nhận giá trị $\left \lfloor \frac{n+2}{3} \right \rfloor$ và $\left \lfloor \frac{n+5}{3} \right \rfloor$ thì $m$ nhận giá trị $2$ và $3$
+ Khi $p\in\left \{ 2,5,8,11 \right \}$ và $a$ lần lượt nhận giá trị $\left \lfloor \frac{n+2}{3} \right \rfloor$ và $\left \lfloor \frac{n+5}{3} \right \rfloor$ thì $m$ nhận giá trị $1$ và $3$
Từ các nhận xét trên, dễ dàng tính được tổng số nghiệm nguyên dương $s$ thỏa mãn các điều kiện đề bài :
$s=\left\{\begin{matrix}3k^2\ neu\ p=0\\3k^2+2k\ neu\ p\in\left \{ 1,4 \right \}\\3k^2+k\ neu\ p=2\\3k^2+3k+1\ neu\ p\in\left \{ 3,6 \right \}\\3k^2+4k+1\ neu\ p\in\left \{ 5,8 \right \}\\3k^2+5k+2\ neu\ p\in\left \{ 7,10 \right \}\\3k^2+6k+3\ neu\ p=9\\3k^2+7k+1\ neu\ p=11 \end{matrix}\right.$

wait...Theo anh @perfectstrong:
$s=\begin{cases}
3k^2\ &\text{nếu $p=0,$}\\
3k^2+2k &\text {nếu $\,p\in\left \{ 1,4 \right \},$}\\
3k^2+k&\text {nếu $p=2,$}\\
3k^2+3k+1&\text {nếu $p\in\left \{ 3,6 \right \},$}\\
3k^2+4k+1&\text {nếu $p\in\left \{ 5,8 \right \},$}\\
3k^2+5k+2&\text {nếu $p\in\left \{ 7,10 \right \},$}\\
3k^2+6k+3 &\text {nếu $p=9,$}\\
3k^2+7k+1 &\text {nếu $p=11.$}
\end{cases}$

Bài viết đã được chỉnh sửa nội dung bởi Nobodyv3: 01-01-2023 - 12:05

===========
Thà rót cho ta..... trăm nghìn chung... rượu độc ...miễn sao đừng bắt em làm toán!..hu hu...

#4
Nobodyv3

Nobodyv3

    Generating Functions Faithful

  • Thành viên
  • 937 Bài viết
1/ Lập hàm sinh $f(x)$ mà hệ số của $x^n$ là số nghiệm cần tìm. Ta có :
$$f(x)=\sum_{{a,b,c\ge 1\atop a\ge b \ge c }\atop a < b+c}x^{a+b+c}$$
Đặt :
$\begin {align*}
c=y_3+1 \quad\quad y_3\ge 0\\
b=y_2+c \quad\quad y_2\ge 0\\
a=y_1+b\quad\quad y_3\ge 0
\end {align*}$
Do $a<b+c$ nên :
$\begin {align*}
y_1+y_2+y_3+1&<(y_2+y_3+1)+(y_3+1)\\
y_1&<y_3+1\\
y_1&\leq y_3\\
a+b+c&=(y_1+y_2+y_3+1)+(y_2+y_3+1)+(y_3+1)\\
&=y_1+2y_2+3y_3+3\\
\Longrightarrow f(x)=\sum_{y_1,y_2,y_3\geq 0\atop y_1\leq y_3}x^{y_1+2y_2+3y_3+3}
\end {align*}$
Từ $y_1\leq y_3:$
$\begin {align*}
y_3&=t_3+y_1\quad\quad t_3\geq0\\
y_2&=t_2\quad\quad\quad t_2\geq0\\
y_1&=t_1\quad\quad\quad t_1\geq0\\
\Longrightarrow y_1+2y_2+3y_3+3&=t_1+2t_2+3(t_3+t_1)+3\\
&=4t_1+2t_2+3t_3+3\\
\Longrightarrow f(x)&=x^3\sum_{t_1,t_2,t_3\geq0}x^{4t_1+2t_2+3t_3}\\
&=x^3\sum_{t_2\geq0}x^{2t_2}\sum_{t_3\geq0}x^{3t_3}\sum_{t_1\geq0}x^{4t_1}\\
&=\boldsymbol {\frac {x^3}{(1-x^2)(1-x^3)(1-x^4)}}
\end{align*}$
Tách phân thức :
$\begin {align*}
\frac{1}{(1-x^2)(1-x^3)(1-x^4)}&=\frac{7}{32}\cdot \frac{1}{1+x}+\frac{1}{16}\cdot \frac{1}{(1+x)^2}\\
&+\frac{59}{288}\cdot\frac{1}{1-x}+\frac{1}{8}\cdot\frac{1}{(1-x)^2}+\frac{1}{24}\cdot\frac{1}{(1-x)^3}\\
&+\left(\frac{1}{8}-\frac{x}{8}\right )\cdot\frac{1}{1+x^2}+\left(\frac{2}{9}+\frac{x}{9}\right )\cdot\frac{1}{1+x+x^2}\\
\Longrightarrow \left [ x^n \right ]f(x)&=\left [ x^{n-3} \right ]\left( {\frac{7}{32}\sum_{k\geq0}(-1)^k x^k+\frac {1}{16}\sum_{k\geq0}(k+1)(-1)^kx^k} \right.\\
&+\frac {59}{288}\sum_{k\geq0}x^k+\frac {1}{8}\sum_{k\geq0}(k+1)x^k+\frac {1}{24}\sum_{k\geq0}\binom {k+2}{2}x^k\\
&\left. {+\left ( \frac {1}{8}-\frac{x}{8} \right )\sum_{k\geq0}(-1)^kx^{2k}+\left ( \frac {2}{9}+\frac {x}{9} \right)(1-x)\sum_{k\geq0}x^{3k}}\right)
\end {align*}$

Bài viết đã được chỉnh sửa nội dung bởi Nobodyv3: 03-01-2023 - 07:38
LaTeX

===========
Thà rót cho ta..... trăm nghìn chung... rượu độc ...miễn sao đừng bắt em làm toán!..hu hu...

#5
perfectstrong

perfectstrong

    $LOVE(x)|_{x =\alpha}^\Omega=+\infty$

  • Quản lý Toán Ứng dụng
  • 4995 Bài viết

wait...Theo anh @perfectstrong:
$s=\begin{cases}
3k^2\ &\text{nếu $p=0,$}\\
3k^2+2k &\text {nếu $\,p\in\left \{ 1,4 \right \},$}\\
3k^2+k&\text {nếu $p=2,$}\\
3k^2+3k+1&\text {nếu $p\in\left \{ 3,6 \right \},$}\\
3k^2+4k+1&\text {nếu $p\in\left \{ 5,8 \right \},$}\\
3k^2+5k+2&\text {nếu $p\in\left \{ 7,10 \right \},$}\\
3k^2+6k+3 &\text {nếu $p=9,$}\\
3k^2+7k+1 &\text {nếu $p=11.$}
\end{cases}$

Sao lại có tên mình thế :D


Luôn yêu để sống, luôn sống để học toán, luôn học toán để yêu!!! :D
$$\text{LOVE}\left( x \right)|_{x = \alpha}^\Omega = + \infty $$
I'm still there everywhere.

#6
Nobodyv3

Nobodyv3

    Generating Functions Faithful

  • Thành viên
  • 937 Bài viết

Sao lại có tên mình thế :D

Cám ơn anh.
Nhắc đến anh vì anh thường sửa giúp những lỗi như thế này ạ.
===========
Thà rót cho ta..... trăm nghìn chung... rượu độc ...miễn sao đừng bắt em làm toán!..hu hu...

#7
Nobodyv3

Nobodyv3

    Generating Functions Faithful

  • Thành viên
  • 937 Bài viết

1/ Lập hàm sinh $f(x)$ mà hệ số của $x^n$ là số nghiệm cần tìm. Ta có :
$$f(x)=\sum_{{a,b,c\ge 1\atop a\ge b \ge c }\atop a < b+c}x^{a+b+c}$$
Đặt :
$\begin {align*}
c=y_3+1 \quad\quad y_3\ge 0\\
b=y_2+c \quad\quad y_2\ge 0\\
a=y_1+b\quad\quad y_3\ge 0
\end {align*}$
Do $a<b+c$ nên :
$\begin {align*}
y_1+y_2+y_3+1&<(y_2+y_3+1)+(y_3+1)\\
y_1&<y_3+1\\
y_1&\leq y_3\\
a+b+c&=(y_1+y_2+y_3+1)+(y_2+y_3+1)+(y_3+1)\\
&=y_1+2y_2+3y_3+3\\
\Longrightarrow f(x)=\sum_{y_1,y_2,y_3\geq 0\atop y_1\leq y_3}x^{y_1+2y_2+3y_3+3}
\end {align*}$
Từ $y_1\leq y_3:$
$\begin {align*}
y_3&=t_3+y_1\quad\quad t_3\geq0\\
y_2&=t_2\quad\quad\quad t_2\geq0\\
y_1&=t_1\quad\quad\quad t_1\geq0\\
\Longrightarrow y_1+2y_2+3y_3+3&=t_1+2t_2+3(t_3+t_1)+3\\
&=4t_1+2t_2+3t_3+3\\
\Longrightarrow f(x)&=x^3\sum_{t_1,t_2,t_3\geq0}x^{4t_1+2t_2+3t_3}\\
&=x^3\sum_{t_2\geq0}x^{2t_2}\sum_{t_3\geq0}x^{3t_3}\sum_{t_1\geq0}x^{4t_1}\\
&=\boldsymbol {\frac {x^3}{(1-x^2)(1-x^3)(1-x^4)}}
\end{align*}$
Tách phân thức :
$\begin {align*}
\frac{1}{(1-x^2)(1-x^3)(1-x^4)}&=\frac{7}{32}\cdot \frac{1}{1+x}+\frac{1}{16}\cdot \frac{1}{(1+x)^2}\\
&+\frac{59}{288}\cdot\frac{1}{1-x}+\frac{1}{8}\cdot\frac{1}{(1-x)^2}+\frac{1}{24}\cdot\frac{1}{(1-x)^3}\\
&+\left(\frac{1}{8}-\frac{x}{8}\right )\cdot\frac{1}{1+x^2}+\left(\frac{2}{9}+\frac{x}{9}\right )\cdot\frac{1}{1+x+x^2}\\
\Longrightarrow \left [ x^n \right ]f(x)&=\left [ x^{n-3} \right ]\left( {\frac{7}{32}\sum_{k\geq0}(-1)^k x^k+\frac {1}{16}\sum_{k\geq0}(k+1)(-1)^kx^k} \right.\\
&+\frac {59}{288}\sum_{k\geq0}x^k+\frac {1}{8}\sum_{k\geq0}(k+1)x^k+\frac {1}{24}\sum_{k\geq0}\binom {k+2}{2}x^k\\
&\left. {+\left ( \frac {1}{8}-\frac{x}{8} \right )\sum_{k\geq0}(-1)^kx^{2k}+\left ( \frac {2}{9}+\frac {x}{9} \right)(1-x)\sum_{k\geq0}x^{3k}}\right)
\end {align*}$

Còn nữa, rút gọn lại... nhưng em oải quá...
===========
Thà rót cho ta..... trăm nghìn chung... rượu độc ...miễn sao đừng bắt em làm toán!..hu hu...

#8
hxthanh

hxthanh

    Tín đồ $\sum$

  • Hiệp sỹ
  • 3921 Bài viết

wait...Theo anh @perfectstrong:
$s=\begin{cases}
3k^2\ &\text{nếu $p=0,$}\\
3k^2+2k &\text {nếu $\,p\in\left \{ 1,4 \right \},$}\\
3k^2+k&\text {nếu $p=2,$}\\
3k^2+3k+1&\text {nếu $p\in\left \{ 3,6 \right \},$}\\
3k^2+4k+1&\text {nếu $p\in\left \{ 5,8 \right \},$}\\
3k^2+5k+2&\text {nếu $p\in\left \{ 7,10 \right \},$}\\
3k^2+6k+3 &\text {nếu $p=9,$}\\
3k^2+7k+1 &\text {nếu $p=11.$}
\end{cases}$

Với $n=12k+p$
Bạn nào có thể đưa $12$ trường hợp trên về một công thức duy nhất (theo modulo $12$) không?

#9
Nobodyv3

Nobodyv3

    Generating Functions Faithful

  • Thành viên
  • 937 Bài viết
Welcome back!
Case này khó à nha...
===========
Thà rót cho ta..... trăm nghìn chung... rượu độc ...miễn sao đừng bắt em làm toán!..hu hu...

#10
chanhquocnghiem

chanhquocnghiem

    Thiếu tá

  • Thành viên
  • 2494 Bài viết

Với $n=12k+p$
Bạn nào có thể đưa $12$ trường hợp trên về một công thức duy nhất (theo modulo $12$) không?

Với $n=12k+p$.

Đặt $q=2p-3\left \lfloor \frac{p}{2} \right \rfloor$

Khi đó ta có $s=3k^2+qk+\left \lfloor \frac{q}{3} \right \rfloor+\left \lfloor \frac{q}{5} \right \rfloor+\left \lfloor \frac{q}{7} \right \rfloor$.


...

Ðêm nay tiễn đưa

Giây phút cuối vẫn còn tay ấm tay
Mai sẽ thấm cơn lạnh khi gió lay
Và những lúc mưa gọi thương nhớ đầy ...

 

http://www.wolframal...-15)(x^2-8x+12)


#11
Nobodyv3

Nobodyv3

    Generating Functions Faithful

  • Thành viên
  • 937 Bài viết
Welcome back, anh Chanhquocnghiem.
===========
Thà rót cho ta..... trăm nghìn chung... rượu độc ...miễn sao đừng bắt em làm toán!..hu hu...

#12
hxthanh

hxthanh

    Tín đồ $\sum$

  • Hiệp sỹ
  • 3921 Bài viết

Với $n=12k+p$.
Đặt $q=2p-3\left \lfloor \frac{p}{2} \right \rfloor$
Khi đó ta có $s=3k^2+qk+\left \lfloor \frac{q}{3} \right \rfloor+\left \lfloor \frac{q}{5} \right \rfloor+\left \lfloor \frac{q}{7} \right \rfloor$.

Có vẻ như $p=11$ thì công thức không còn đúng
Khi đó công thức này sẽ cho kq $3k^2+7k+4$ thay vì đáp án là $3k^2+7k+1$
11 trường hợp khác thì chính xác!

#13
hxthanh

hxthanh

    Tín đồ $\sum$

  • Hiệp sỹ
  • 3921 Bài viết

Welcome back!
Case này khó à nha...

Bài toán của em rất hay! Có thể phát biểu thành:
Có bao nhiêu tam giác có chu vi là số nguyên dương $n$ mà độ dài các cạnh đều là số nguyên

#14
chanhquocnghiem

chanhquocnghiem

    Thiếu tá

  • Thành viên
  • 2494 Bài viết

Có vẻ như $p=11$ thì công thức không còn đúng
Khi đó công thức này sẽ cho kq $3k^2+7k+4$ thay vì đáp án là $3k^2+7k+1$
11 trường hợp khác thì chính xác!

Khi $p=11$ thì đáp án đúng là $s=3k^2+7k+4$ (Xin xem #2 ở trên)
 


...

Ðêm nay tiễn đưa

Giây phút cuối vẫn còn tay ấm tay
Mai sẽ thấm cơn lạnh khi gió lay
Và những lúc mưa gọi thương nhớ đầy ...

 

http://www.wolframal...-15)(x^2-8x+12)


#15
hxthanh

hxthanh

    Tín đồ $\sum$

  • Hiệp sỹ
  • 3921 Bài viết

Với $n=12k+p$.
Đặt $q=2p-3\left \lfloor \frac{p}{2} \right \rfloor$
Khi đó ta có $s=3k^2+qk+\left \lfloor \frac{q}{3} \right \rfloor+\left \lfloor \frac{q}{5} \right \rfloor+\left \lfloor \frac{q}{7} \right \rfloor$.

Vậy đây là một đáp án hoàn hảo! Bạn có thể diễn đạt “quy trình” tìm ra nó cho mọi người học hỏi được không?

#16
chanhquocnghiem

chanhquocnghiem

    Thiếu tá

  • Thành viên
  • 2494 Bài viết

Trước hết ta đặt $s=3k^2+qk+t$.

Lập bảng :           $p$           $q$           $t$

                            $0$           $0$           $0$

                            $1$           $2$           $0$

                            $2$           $1$           $0$

                            $3$           $3$           $1$

                            $4$           $2$           $0$

                            $5$           $4$           $1$

                            $6$           $3$           $1$

                            $7$           $5$           $2$

                            $8$           $4$           $1$

                            $9$           $6$           $3$

                          $10$           $5$           $2$

                          $11$           $7$           $4$

- Tìm biểu thức của $q$ :

  Nhận xét : Với $2$ trường hợp khởi tạo $p=0$ và $p=1$, ta lần lượt có $q=0$ và $q=2$.

  Do đó có thể nói nếu $p\equiv r\ (mod\ 2)$ thì $q=2r$ hay $q=2\left ( p-2\left \lfloor \frac{p}{2} \right \rfloor \right )$ (công thức này chỉ đúng với 2 dòng khởi tạo)

  Khi $p$ tăng thêm $2$ thì $q$ tăng thêm $1$, do đó biểu thức của $q$ (đúng với cả $12$ dòng) là :

  $q=2\left ( p-2\left \lfloor \frac{p}{2} \right \rfloor \right )+\left \lfloor \frac{p}{2} \right \rfloor=2p-3\left \lfloor \frac{p}{2} \right \rfloor$

- Tìm biểu thức của $t$ :

  Dễ thấy với $7$ dòng đầu ($p$ từ $0$ đến $6$) ta có $t=\left \lfloor \frac{q}{3} \right \rfloor$

  Để biểu thức đúng đến dòng thứ $11$, cần phải thêm số hạng $\left \lfloor \frac{q}{5} \right \rfloor$

  Và muốn cho dòng thứ $12$ biểu thức vẫn đúng thì phải thêm $\left \lfloor \frac{q}{7} \right \rfloor$

  Cuối cùng ta có $t=\left \lfloor \frac{q}{3} \right \rfloor+\left \lfloor \frac{q}{5} \right \rfloor+\left \lfloor \frac{q}{7} \right \rfloor$.


...

Ðêm nay tiễn đưa

Giây phút cuối vẫn còn tay ấm tay
Mai sẽ thấm cơn lạnh khi gió lay
Và những lúc mưa gọi thương nhớ đầy ...

 

http://www.wolframal...-15)(x^2-8x+12)


#17
chuyenndu

chuyenndu

    Trung sĩ

  • Thành viên
  • 178 Bài viết

đặt n=2m (n lẻ làm tương tự)

số bộ (a,b,c) sao cho $1\le c\le b\le a\le 2m$ là $C_{2m+2}^3$

số cặp (b,c) sao cho b+c=k và $1\le c\le b$ là $\left\lfloor\dfrac{k}{2}\right\rfloor$, nên số bộ (a,b,c) mà $a\ge b\ge c$ và $b+c\le a$ bằng
$$\sum_{a=1}^{2m} \sum_{k=1}^{a} \left\lfloor\dfrac{k}{2}\right\rfloor=\sum_{a=1}^{2m}\left\lfloor\dfrac{a^2}{4}\right\rfloor=\dfrac{m(m+1)(4m-1)}{6}$$
vậy số bộ cần tính bằng $C_{2m+2}^3-\dfrac{m(m+1)(4m-1)}{6}$



#18
chanhquocnghiem

chanhquocnghiem

    Thiếu tá

  • Thành viên
  • 2494 Bài viết

đặt n=2m (n lẻ làm tương tự)

số bộ (a,b,c) sao cho $1\le c\le b\le a\le 2m$ là $C_{2m+2}^3$

số cặp (b,c) sao cho b+c=k và $1\le c\le b$ là $\left\lfloor\dfrac{k}{2}\right\rfloor$, nên số bộ (a,b,c) mà $a\ge b\ge c$ và $b+c\le a$ bằng
$$\sum_{a=1}^{2m} \sum_{k=1}^{a} \left\lfloor\dfrac{k}{2}\right\rfloor=\sum_{a=1}^{2m}\left\lfloor\dfrac{a^2}{4}\right\rfloor=\dfrac{m(m+1)(4m-1)}{6}$$
vậy số bộ cần tính bằng $C_{2m+2}^3-\dfrac{m(m+1)(4m-1)}{6}$

Vậy nếu $n=10$ (tức là $m=5$) thì theo đáp án của bạn, số bộ thỏa mãn sẽ là $C_{12}^3-\frac{5.6.19}{6}=125$.

Làm sao mà có đến $125$ bộ, mình thấy chỉ có $2$ bộ thỏa mãn thôi !
 


...

Ðêm nay tiễn đưa

Giây phút cuối vẫn còn tay ấm tay
Mai sẽ thấm cơn lạnh khi gió lay
Và những lúc mưa gọi thương nhớ đầy ...

 

http://www.wolframal...-15)(x^2-8x+12)


#19
hxthanh

hxthanh

    Tín đồ $\sum$

  • Hiệp sỹ
  • 3921 Bài viết

đặt n=2m (n lẻ làm tương tự)
số bộ (a,b,c) sao cho $1\le c\le b\le a\le 2m$ là $C_{2m+2}^3$
số cặp (b,c) sao cho b+c=k và $1\le c\le b$ là $\left\lfloor\dfrac{k}{2}\right\rfloor$, nên số bộ (a,b,c) mà $a\ge b\ge c$ và $b+c\le a$ bằng
$$\sum_{a=1}^{2m} \sum_{k=1}^{a} \left\lfloor\dfrac{k}{2}\right\rfloor=\sum_{a=1}^{2m}\left\lfloor\dfrac{a^2}{4}\right\rfloor=\dfrac{m(m+1)(4m-1)}{6}$$
vậy số bộ cần tính bằng $C_{2m+2}^3-\dfrac{m(m+1)(4m-1)}{6}$

Bạn quên mất một điều kiện rất quan trọng đó là $a+b+c=2m$
Hay $a+k=2m$. Như vậy hai biến chạy này ràng buộc và không thể chạy “tự do” như cách bạn lấy tổng được!

@chanhquocnghiem
Cái $q$ nói dễ nhận ra còn có lý, vì các số hạng xen kẽ là dãy số tự nhiên tăng dần.
Còn $t$ mà “dễ thấy” thì quả là tinh tế!

#20
hxthanh

hxthanh

    Tín đồ $\sum$

  • Hiệp sỹ
  • 3921 Bài viết
Số các tam giác cạnh nguyên có chu vi $n$ là $S_n$:
$$S_n=\left\lfloor \frac{n^2+6}{12}\right\rfloor-\left\lfloor \frac{n}{4} \right\rfloor \left\lfloor \frac{n+2}{4}\right\rfloor$$
Các bạn thử “check” xem kết quả này có trùng với kết quả bạn chanhquocnghiem tìm được không nhé!




1 người đang xem chủ đề

0 thành viên, 1 khách, 0 thành viên ẩn danh